Skip to content

Commit c385a5d

Browse files
committed
update
1 parent 8c4539f commit c385a5d

17 files changed

+301
-68
lines changed

.DS_Store

8 KB
Binary file not shown.

AlgebraicGeometry/.DS_Store

6 KB
Binary file not shown.

AlgebraicGeometry/1/.DS_Store

6 KB
Binary file not shown.

AlgebraicGeometry/1/solution/AlgebraicGeometry1.tex

+59-59
Original file line numberDiff line numberDiff line change
@@ -10,102 +10,102 @@
1010
\large
1111
\setlength{\baselineskip}{1.2em}
1212
\ifpreface
13-
\input{../../../global/preface}
14-
\newgeometry{left=2cm,right=2cm,top=2cm,bottom=2cm}
13+
\input{../../../global/preface}
14+
\newgeometry{left=2cm,right=2cm,top=2cm,bottom=2cm}
1515
\else
16-
\newgeometry{left=2cm,right=2cm,top=2cm,bottom=2cm}
17-
\maketitle
16+
\newgeometry{left=2cm,right=2cm,top=2cm,bottom=2cm}
17+
\maketitle
1818
\fi
1919
%from_here_to_type
2020
\newcommand{\A}{\mathbbm{A}}
2121
\begin{problem}
22-
$P$ is an ideal of a unitary commutative ring $A$, then $P$ is prime ideal of $A\iff A/P$ is integral domain.
22+
$P$ is an ideal of a unitary commutative ring $A$, then $P$ is prime ideal of $A\iff A/P$ is integral domain.
2323
\end{problem}
2424
\(\)
2525
\begin{solution}
26-
$\Rightarrow $:
26+
$\Rightarrow $:
2727

28-
Since $A$ is a unitary commutative ring, so $A/P$ is unitary commutative ring, too. So we only need to prove $[ab]=[0]\Rightarrow [a]=[0]\xor [b]=[0]$.
29-
Obviously $[ab]=0\iff ab\in P\iff a\in P\xor b\in P\iff [a]=[0]\xor [b]=[0]$.
28+
Since $A$ is a unitary commutative ring, so $A/P$ is unitary commutative ring, too. So we only need to prove $[ab]=[0]\Rightarrow [a]=[0]\xor [b]=[0]$.
29+
Obviously $[ab]=0\iff ab\in P\iff a\in P\xor b\in P\iff [a]=[0]\xor [b]=[0]$.
3030

31-
$\Leftarrow $:
31+
$\Leftarrow $:
3232

33-
As the same, $ab\in P\iff [ab]=[0]\Rightarrow [a]=[0]\xor [b]=[0]\iff a\in P\xor b\in P$, so $P$ is prime ideal.
33+
As the same, $ab\in P\iff [ab]=[0]\Rightarrow [a]=[0]\xor [b]=[0]\iff a\in P\xor b\in P$, so $P$ is prime ideal.
3434
\end{solution}
3535

3636
\begin{problem}
37-
$M$ is an ideal of a unitary commutative ring $A$, then $M$ is maximal ideal of $A \iff A/M$ is a field.
37+
$M$ is an ideal of a unitary commutative ring $A$, then $M$ is maximal ideal of $A \iff A/M$ is a field.
3838
\end{problem}
3939
\begin{solution}
40-
$\Rightarrow $:
40+
$\Rightarrow $:
4141

42-
Consider $[a]\in A/M\minus [0]$, we will prove it has a reverse. Consider $N:=\{xm+ya:x,y\in A,m\in M\}$ is the minimum ideal of $A$ contains $M$ and $a$.
43-
Since $[a]\neq [0]$ we know $a\notin M$, so $M\subsetneqq N$.
44-
Noting $M$ is maximal, so $N=A$. That means $\exists x,y\in A,m\in M,xm+ya=1$. So $[xm+ya]=[1]$. Since $[xm]=[0]$ we get $[y][a]=1$, i.e., $[y]=[a]^{-1}$.
42+
Consider $[a]\in A/M\minus [0]$, we will prove it has a reverse. Consider $N:=\{xm+ya:x,y\in A,m\in M\}$ is the minimum ideal of $A$ contains $M$ and $a$.
43+
Since $[a]\neq [0]$ we know $a\notin M$, so $M\subsetneqq N$.
44+
Noting $M$ is maximal, so $N=A$. That means $\exists x,y\in A,m\in M,xm+ya=1$. So $[xm+ya]=[1]$. Since $[xm]=[0]$ we get $[y][a]=1$, i.e., $[y]=[a]^{-1}$.
4545

46-
$\Leftarrow $:
46+
$\Leftarrow $:
4747

48-
Consider $a\in A\minus M, N:=\{xp+ya:x,y\in A,p\in P\}$, we will prove $N=A$, which means $M$ is maximal.
49-
Since $A/M$ is field, $\exists y\in A, [y]=[a]^{-1}$. That's means $ay-1\in M\subset N$. Noting $ay\in N$, so $1\in N$, thus $N=A$.
48+
Consider $a\in A\setminus M, N:=\{xp+ya:x,y\in A,p\in P\}$, we will prove $N=A$, which means $M$ is maximal.
49+
Since $A/M$ is field, $\exists y\in A, [y]=[a]^{-1}$. That's means $ay-1\in M\subset N$. Noting $ay\in N$, so $1\in N$, thus $N=A$.
5050
\end{solution}
5151

5252
\begin{problem}
53-
A ring $A$ is noetherian, $I\subset A$ is an ideal of $A$, then $A/I$ is noetherian.
53+
A ring $A$ is noetherian, $I\subset A$ is an ideal of $A$, then $A/I$ is noetherian.
5454
\end{problem}
5555
\begin{solution}
56-
Consider an ideal $J\subset A/I$, let $M:=\{x\in A:[x]\in J\}$. Then $\forall a\in A,x\in M,[ax]=[a][x]\in J$, so $ax\in M$. $\forall a,b\in M,[a-b]=[a]-[b]\in J$, so $a-b\in M$. So $M$ is an ideal of $A$.
57-
Since $A$ is noetherian, we can assume $M=(f_i,i=1,2,\cdots n)$. Now we will prove $J=([f_i],i=1,2,\cdots n)$.
58-
Consider $[f]\in J$, from definition of $M$ we know $f\in M$, so $f=\sum_{i=1}^na_if_i,a_i\in A$, thus $[f]=\left[\sum_{i=1}^na_if_i\right]=\sum_{i=1}^n[a_i][f_i]$. So $J=([f_i],i=1,2,\cdots n)$.
56+
Consider an ideal $J\subset A/I$, let $M:=\{x\in A:[x]\in J\}$. Then $\forall a\in A,x\in M,[ax]=[a][x]\in J$, so $ax\in M$. $\forall a,b\in M,[a-b]=[a]-[b]\in J$, so $a-b\in M$. So $M$ is an ideal of $A$.
57+
Since $A$ is noetherian, we can assume $M=(f_i,i=1,2,\cdots n)$. Now we will prove $J=([f_i],i=1,2,\cdots n)$.
58+
Consider $[f]\in J$, from definition of $M$ we know $f\in M$, so $f=\sum_{i=1}^na_if_i,a_i\in A$, thus $[f]=\left[\sum_{i=1}^na_if_i\right]=\sum_{i=1}^n[a_i][f_i]$. So $J=([f_i],i=1,2,\cdots n)$.
5959
\end{solution}
6060

6161
\begin{problem}
62-
\label{pro:4}
63-
$K$ is a field, $A=K[x_1,x_2,\cdots x_n],\A^n_K=K^n$. For ideal $I$ of $A$ let $V(I):=\{p\in\A_K^n:f(p)=0,\forall f\in I\}$. Then $V(I_1)\cup V(I_2)=V(I_1\cap I_2)=V(I_1I_2)$
62+
\label{pro:4}
63+
$K$ is a field, $A=K[x_1,x_2,\cdots x_n],\A^n_K=K^n$. For ideal $I$ of $A$ let $V(I):=\{p\in\A_K^n:f(p)=0,\forall f\in I\}$. Then $V(I_1)\cup V(I_2)=V(I_1\cap I_2)=V(I_1I_2)$
6464
\end{problem}
6565
\begin{solution}
66-
\begin{lemma}
67-
\label{lem:1}
68-
$I\subset J\Rightarrow V(I)\supset V(J)$.
69-
\end{lemma}
70-
\begin{proof}
71-
Consider $p\in V(J)$, we get $\forall f\in J,f(p)=0$. Since $I\subset J$, so $\forall f\in I,f(p)=0$, i.e., $f\in V(I)$.
72-
\end{proof}
73-
From Lemma \Cref{lem:1} we know $V(I_1\cap I_2)\subset V(I_1I_2)$, so we only need to prove $V(I_1)\cup V(I_2)\subset V(I_1\cap I_2),V(I_1I_2)\subset V(I_1)\cup V(I_2)$.
74-
\begin{itemize}
75-
\item $V(I_1)\cup V(I_2)\subset V(I_1\cap I_2)$:
76-
From \Cref{lem:1} It's obvious.
77-
\item $V(I_1I_2)\subset V(I_1)\cup V(I_2)$:
78-
Consider $p\in V(I_1I_2)$.
79-
If $p\notin V(I_1)\cup V(I_2)$, then $\exists f_1\in I_1,f_2\in I_2, f_1(p)\neq 0,f_2(p)\neq 0$. Now consider $f=f_1f_2\in I_1I_2$, we get $f(p)=f_1(p)f_2(p)\neq 0$, so $p\notin V(I_1I_2)$, it's a contradiction.
80-
\end{itemize}
66+
\begin{lemma}
67+
\label{lem:1}
68+
$I\subset J\Rightarrow V(I)\supset V(J)$.
69+
\end{lemma}
70+
\begin{proof}
71+
Consider $p\in V(J)$, we get $\forall f\in J,f(p)=0$. Since $I\subset J$, so $\forall f\in I,f(p)=0$, i.e., $f\in V(I)$.
72+
\end{proof}
73+
From Lemma \Cref{lem:1} we know $V(I_1\cap I_2)\subset V(I_1I_2)$, so we only need to prove $V(I_1)\cup V(I_2)\subset V(I_1\cap I_2),V(I_1I_2)\subset V(I_1)\cup V(I_2)$.
74+
\begin{itemize}
75+
\item $V(I_1)\cup V(I_2)\subset V(I_1\cap I_2)$:
76+
From \Cref{lem:1} It's obvious.
77+
\item $V(I_1I_2)\subset V(I_1)\cup V(I_2)$:
78+
Consider $p\in V(I_1I_2)$.
79+
If $p\notin V(I_1)\cup V(I_2)$, then $\exists f_1\in I_1,f_2\in I_2, f_1(p)\neq 0,f_2(p)\neq 0$. Now consider $f=f_1f_2\in I_1I_2$, we get $f(p)=f_1(p)f_2(p)\neq 0$, so $p\notin V(I_1I_2)$, it's a contradiction.
80+
\end{itemize}
8181
\end{solution}
8282

8383
\begin{problem}
84-
$K$ is an infinite field, then $\A_k^n$ is not Hausdorff.
84+
$K$ is an infinite field, then $\A_k^n$ is not Hausdorff.
8585
\end{problem}
8686
\begin{solution}
87-
\begin{lemma}
88-
\label{lem:2}
89-
$K$ is an infinite field, $f\in K[x_1,x_2,\cdots x_n]\setminus \{0\}$, then exists $p\in \A_K^n,f(p)\neq 0$.
90-
\end{lemma}
91-
\begin{proof}
92-
Use MI. When $n=0,K[x_1,x_2,\cdots x_n]=K$, so it's obvious. Assume for $n=k$ it's right, when goes to $k+1$:
87+
\begin{lemma}
88+
\label{lem:2}
89+
$K$ is an infinite field, $f\in K[x_1,x_2,\cdots x_n]\setminus \{0\}$, then exists $p\in \A_K^n,f(p)\neq 0$.
90+
\end{lemma}
91+
\begin{proof}
92+
Use MI. When $n=0,K[x_1,x_2,\cdots x_n]=K$, so it's obvious. Assume for $n=k$ it's right, when goes to $k+1$:
9393

94-
Consider $h\in K(x_1,x_2,\cdots x_k)[x_{k+1}],h(x_{k+1}):=f(x_1,x_2,\cdots x_k,x_{k+1})$ is a non-zero polynomial so it has finite root. So exists $a\in K,h(a)\neq 0$. So $g:=f(x_1,x_2,\cdots x_k,a)\in K[x_1,x_2,\cdots x_k]\neq 0$.
95-
By induction hypothesis we get $\exists b\in \A_K^k,g(b)\neq 0$. Let $p:=(b,a)\in \A_K^{k+1}$, then $f(p)\neq 0$.
96-
\end{proof}
97-
In fact, it's not only not Hausdorff, it's kind of ''absolutly not Hausdorff'' because every pair of point can't be seperated.
98-
Consider two point $p\neq q,p,q\in \A_K^n$.
99-
Assume $p=(p_1,p_2,\cdots p_n),q=(q_1,q_2,\cdots q_n),p_1\neq q_1$. Assume two open set $V(I_1)^c,V(I_2)^c$ can seperate $p,q$, then $V(I_1)\cup V(I_2)=\A_K^n$.
100-
From \Cref{pro:4} we know $V(I_1I_2)=\A_K^n$. So $\forall f\in V(I_1I_2),\forall p\in \A_K^n,f(p)=0$.
101-
Then from Lemma\Cref{lem:2} we can get $f=0$. So $I_1I_2=\{0\}$.
102-
Since $p\notin V(I_1),q\notin V(I_2)$, we know $I_1,I_2\neq \{0\}$. So $\exists f_1\in I_1\neq 0,\exists f_2\in I_2\neq 0$, and thus $f=f_1f_2\in I_1I_2\neq 0$, contradiction!
103-
\(\phi\)
104-
So there is not a pair of points can be seperated by two disjoint open set.
94+
Consider $h\in K(x_1,x_2,\cdots x_k)[x_{k+1}],h(x_{k+1}):=f(x_1,x_2,\cdots x_k,x_{k+1})$ is a non-zero polynomial so it has finite root. So exists $a\in K,h(a)\neq 0$. So $g:=f(x_1,x_2,\cdots x_k,a)\in K[x_1,x_2,\cdots x_k]\neq 0$.
95+
By induction hypothesis we get $\exists b\in \A_K^k,g(b)\neq 0$. Let $p:=(b,a)\in \A_K^{k+1}$, then $f(p)\neq 0$.
96+
\end{proof}
97+
In fact, it's not only not Hausdorff, it's kind of ''absolutly not Hausdorff'' because every pair of point can't be seperated.
98+
Consider two point $p\neq q,p,q\in \A_K^n$.
99+
Assume $p=(p_1,p_2,\cdots p_n),q=(q_1,q_2,\cdots q_n),p_1\neq q_1$. Assume two open set $V(I_1)^c,V(I_2)^c$ can seperate $p,q$, then $V(I_1)\cup V(I_2)=\A_K^n$.
100+
From \Cref{pro:4} we know $V(I_1I_2)=\A_K^n$. So $\forall f\in V(I_1I_2),\forall p\in \A_K^n,f(p)=0$.
101+
Then from Lemma\Cref{lem:2} we can get $f=0$. So $I_1I_2=\{0\}$.
102+
Since $p\notin V(I_1),q\notin V(I_2)$, we know $I_1,I_2\neq \{0\}$. So $\exists f_1\in I_1\neq 0,\exists f_2\in I_2\neq 0$, and thus $f=f_1f_2\in I_1I_2\neq 0$, contradiction!
103+
\(\phi\)
104+
So there is not a pair of points can be seperated by two disjoint open set.
105105
\end{solution}
106106
\begin{figure}
107-
\centering
108-
\includegraphics{~/Pictures/2-3.jpg}%![aaa](~/Pictures/2-3.jpg)
107+
\centering
108+
\includegraphics{~/Pictures/2-3.jpg}%![aaa](~/Pictures/2-3.jpg)
109109

110110
\end{figure}
111111
\end{document}
Binary file not shown.

AlgebraicGeometry/10/solution/AlgebraicGeometry10.tex

+6-8
Original file line numberDiff line numberDiff line change
@@ -11,11 +11,11 @@
1111
\large
1212
\setlength{\baselineskip}{1.2em}
1313
\ifpreface
14-
\input{../../../global/preface}
15-
\newgeometry{left=2cm,right=2cm,top=2cm,bottom=2cm}
14+
\input{../../../global/preface}
15+
\newgeometry{left=2cm,right=2cm,top=2cm,bottom=2cm}
1616
\else
17-
\newgeometry{left=2cm,right=2cm,top=2cm,bottom=2cm}
18-
\maketitle
17+
\newgeometry{left=2cm,right=2cm,top=2cm,bottom=2cm}
18+
\maketitle
1919
\fi
2020
%from_here_to_type
2121
\begin{problem}
@@ -34,9 +34,6 @@
3434
So \(\ker \theta = \ide(V)\). And easily \(\theta\) is surjective, so we get
3535
\(m_p=M_p/\ker \theta=M_p / \ide(V)\).
3636
\end{solution}
37-
\begin{enumerate}
38-
\item
39-
\end{enumerate}
4037

4138
\begin{problem}
4239
Prove that \(M_p/M_p^2+\ide(V) \cong m_p/m_p^2\).
@@ -73,5 +70,6 @@
7370

7471
Obviously \(\tau\) is injective, so it's isomorphic.
7572
\end{solution}
76-
7773
\end{document}
74+
75+

RiemannGeometry/.subject

+1
Original file line numberDiff line numberDiff line change
@@ -0,0 +1 @@
1+
RiemannGeometry

RiemannGeometry/1/.number

+1
Original file line numberDiff line numberDiff line change
@@ -0,0 +1 @@
1+
1
Binary file not shown.
56.4 KB
Binary file not shown.
Original file line numberDiff line numberDiff line change
@@ -0,0 +1,144 @@
1+
%arara: xelatex
2+
%!Mode:: "TeX:UTF-8"
3+
%!TEX TS-program = xelatex
4+
\documentclass{ctexart}
5+
\newif\ifpreface
6+
\prefacetrue
7+
\input{../../../global/all}
8+
\DeclareMathOperator{\sgn}{sgn}
9+
\begin{document}
10+
\large
11+
\setlength{\baselineskip}{1.2em}
12+
\ifpreface
13+
\input{../../../global/preface}
14+
\else
15+
\maketitle
16+
\fi
17+
\newgeometry{left=2cm,right=2cm,top=2cm,bottom=2cm}
18+
\everymath{\displaystyle}
19+
\newlength\inlineHeight
20+
\newlength\inlineWidth
21+
\long\def\(#1\){%
22+
\settoheight{\inlineHeight}{$#1$}%
23+
\settowidth{\inlineWidth}{$#1$}%
24+
\ifdim \inlineWidth > 0.5\textwidth%
25+
$$#1$$%
26+
\else%
27+
\ifdim \inlineHeight > 1.5em%
28+
$$#1$$%
29+
\else%
30+
$#1$%
31+
\fi%
32+
\fi%
33+
}
34+
%from_here_to_type
35+
\begin{problem}\label{pro:1.1}
36+
Assume \(\mathcal{A}_0=\{(U_\alpha,\phi_\alpha):\alpha \in I\}\) is a \(C^r\)-compatible coordinate covery of a \(m\)-dimensional manifold \(M\), let
37+
\(\mathcal{A}:= \{(U,\phi): (U,\phi) \text{ is chart of } M, \AND \forall (V,\psi)\in \mathcal{A}_0,(U,\phi)\text{ is compatible with }(V,\psi)\}\).
38+
Then \(\mathcal{A}\) is unique \(C^r\)-differential structure on \(M\) contains \(\mathcal{A}_0\).
39+
\end{problem}
40+
\begin{solution}
41+
42+
First, easily \(\mathcal{A}_0 \subset \mathcal{A}\) by definition of \(\mathcal{A}_0\).
43+
Now we should prove \(\mathcal{A}\) is differential structure on \(M\). Let \((U,\phi),(V,\psi) \in \mathcal{A}\).
44+
If \((U,\phi)\in \mathcal{A}_0\), then by definition of \(\mathcal{A}_0\) we know \((U,\phi)\) is compatible to \((V,\psi)\).
45+
If \((U,\phi),(V,\psi) \notin \mathcal{A}_0\), then consider \(U\cap V\). If \(U\cap V=\varnothing\), then \((U,\phi)\) is compatible with \((V,\psi)\).
46+
Now assume \(W=U \cap V\neq \varnothing\). Consider \(\gamma:=\psi \circ \phi^{-1} :\phi(W) \to \psi(W)\).
47+
For any \(x \in W\), since \(\mathcal{A}_0\) is covery of \(M\), we know \(\exists (T,\tau) \in \mathcal{A}_0,x \in T\).
48+
Then by the definition of \(\mathcal{A}\), we know \((T,\tau)\) is compatible locally on \(x\) with both \((U,\phi)\) and \((V,\psi)\).
49+
So \((U,\phi)\) is locally compatible with \((V,\psi)\) on \(x\). Since \(x\) is arbitrary, we know \((U,\phi)\) is compatible with \((V,\psi)\).
50+
So \(\mathcal{A}\) is differential structure of \(M\).
51+
52+
Now we assume \(\mathcal{B}\) is another differential structure of \(M\) contains \(\mathcal{A}_0\).
53+
Since \(\mathcal{B}\) is compatible, we get \(\mathcal{B} \subset \mathcal{A}\).
54+
Since \(\mathcal{B}\) is maximal, we get \(\mathcal{B}=\mathcal{A}\).
55+
So \(\mathcal{A}\) is unique.
56+
\end{solution}
57+
\begin{problem}\label{pro:1.2}
58+
Assume \((U,\phi;x^i),(V,\psi;y^i),(W,\chi;z^i)\) are three local coordinate on an \(m\)-dimensional smooth manifold \(M\), and \(W \cap V \cap U \neq \varnothing\).
59+
Prove that on \(\phi(U \cap V \cap W)\), we have:
60+
\(\left( \frac{\partial z^i}{\partial x^j}\right)=\left(\frac{\partial z^i}{\partial y^k}\right)\left(\frac{\partial y^k}{\partial x^j}\right)\).
61+
\end{problem}
62+
\begin{solution}
63+
For fixed \(1 \leq i,j \leq m\), we have \(\frac{\partial z^i}{\partial x^j}=\sum_{k=1}^{m} \frac{\partial z^i}{\partial y^k} \frac{\partial y^k}{\partial x^j}\).
64+
So easily to get that \(\left( \frac{\partial z^i}{\partial x^j}\right)=\left(\frac{\partial z^i}{\partial y^k}\right)\left(\frac{\partial y^k}{\partial x^j}\right)\).
65+
66+
We let \((W,\chi;z^i)=(U,\phi;x^i)\), then we get:
67+
\(I_m=\left(\frac{\partial x^i}{\partial y^k}\right)\left(\frac{\partial y^k}{\partial x^j}\right)\).
68+
So both terms on the right side are invertible, thus non-singular.
69+
\end{solution}
70+
\begin{problem}\label{pro:1.3}
71+
Assume \(M\) is orientable and connected, prove that \(M\) has exactly two different oriention.
72+
\end{problem}
73+
% \begin{lemma}\label{lem:connected_manifold_is_path_connected}
74+
% Assume \(M\) is a connected manifold, then it's path-connected.
75+
% \end{lemma}
76+
% \begin{proof}
77+
% For \(x,y \in M\), we define \(x \sim y\) if and only if there is a path from \(x\) to \(y\).
78+
% Easily \(\sim\) is equivalent relation.
79+
% For \(x \in M\), we define \(U:=\{y \in M: x \sim y\}\). Obviously \(x \in U \neq \varnothing\),
80+
% so we only need to prove \(U\) is both open and closed.
81+
% Assume \(y \in U\), since \(M\) is manifold, we obtain \(\exists V\) is homeomorphic to \(\mathbb{R}^n\), thus path-connected, and \(y \in V\).
82+
% So \(\forall z \in V,z \sim y\), so \(z \sim x\). So \(V \subset U\). So \(U\) is open.
83+
% For \(y \notin U\), samely \(\exists V\) is open and path-connected and \(y \in V\).
84+
% If \(z \in V\) and \(z \sim x\) then we will get \(y \sim x\), contradiction!
85+
% So \(\forall z \in V,z \not \sim x\). So \(V \subset U^c\), so \(U^c\) is open, thus \(U\) is closed.
86+
% Since \(M\) is connected and \(U\) is both open and closed and \(U \neq \varnothing\), we obtain \(U = M\).
87+
% So \(M\) is path-connected.
88+
% \end{proof}
89+
90+
% \begin{lemma}\label{lem:1.3}
91+
% Assume \(\mathcal{A}\) is an oriention of \(M\). Then for every local corrdinate \((V;y^i)\), one of \((V;y^i)\) and \((V;z^i)\) is in \(\mathcal{A}\),
92+
% where \(z^i=
93+
% \begin{cases}
94+
% y^i & i>1\\
95+
% -y^i & i=1
96+
% \end{cases}\).
97+
% \end{lemma}
98+
% \begin{proof}
99+
% Since Jacobi determinant is continious, and Jacobi determinant of transition map is non-zero, so it's always positive or negative.
100+
% Let \(\mathcal{B}:=\{(U;x^i) \in \mathcal{A}:U \cap V \neq \varnothing, J_{x,y}>0\}\) and \(\mathcal{C}:=\{(U;x^i) \in \mathcal{A}:U \cap V \neq \varnothing, J_{x,y}<0\}\).
101+
% If \(\mathcal{C}\) is empty then \((V;y^i)\in \mathcal{A}\) by maximalism of \(\mathcal{A}\).
102+
% If \(\mathcal{B}\) is empty then \((V;z^i)\in \mathcal{A}\) because \(J_{x,y}=-J_{x,z}\).
103+
% So we only need to prove it's impossible that \(\mathcal{B}\) and \(\mathcal{C}\) are both non-empty.
104+
%
105+
% Assume \((U;x^i) \in \mathcal{B}\) and \((T;z^i) \in \mathcal{C}\). By definition of \(\mathcal{B},\mathcal{C}\) we get \(U \cap V \neq \varnothing,T \cap V \neq \varnothing\).
106+
% Let \(p \in U \cap V,q \in T \cap V\). Since \((V;y^i)\) is homeomorphic to a subset of \(\mathbb{R}^n\), we know there is a path from \(p\) to \(q\).
107+
% Easily this path is compact, so \(\exists p_0=p,p_1,p_2,\cdots,p_k=q\) on this path and \(\{(U_i;y_i^j):i=0,\cdots,k\} \subset \mathcal{A}\) is local coordinate of \(p_i\), and this path is in \(\bigcup_{t=0}^{k} U_t\).
108+
% \end{proof}
109+
110+
\begin{solution}
111+
Since \(M\) is orientable, we can assume that \(\mathcal{B} \subset \mathcal{A}\) is an oriention of \(M\), where \(\mathcal{A}\) is all local coordinate of \(M\).
112+
Now consider \(\mathcal{C}:=\{(U;-x^i):(U;x^i) \in \mathcal{B}\}\). Easily to check that \(\mathcal{C}\) is an oriention of \(M\), too.
113+
And obviously \(\mathcal{B} \cap \mathcal{C} = \varnothing\), thus \(\mathcal{B} \neq \mathcal{C}\). So there is two oriention. Now we need to prove there is no other oriention.
114+
115+
Assume \(\mathcal{D}\) is an oriention of \(M\). We will define a function \(\sgn:M \to \{1,-1\}\) by
116+
\(\sgn(p)=1 \iff \exists (U_0,x_0^i) \in \mathcal{B},\exists (V_0,y_0^i) \in \mathcal{D},p \in U_0 \cap V_0,J_{x_0,y_0}(p)>0\).
117+
We will prove that \(\sgn(p)=1 \implies \forall (U,x^i) \in \mathcal{B},\forall (V;y^i) \in \mathcal{D},p \in U \cap V \implies J_{x,y}(p)>0\).
118+
It's easy because \(J_{x,y}=J_{x,x_0}J_{x_0,y_0}J_{y_0,y}\), and \(J_{x,x_0}>0,J_{y,y_0}>0\) by definition of oriention.
119+
So \(\sgn(p)=-1 \iff \exists (U_0,x_0^i) \in \mathcal{C},\exists (V_0,y_0^i) \in \mathcal{D},p \in U_0 \cap V_0,J_{x_0,y_0}(p)>0 \)
120+
\(\iff \forall (U,x^i) \in \mathcal{B},\forall (V;y^i) \in \mathcal{D},p \in U \cap V \implies J_{x,y}(p)>0 \).
121+
Noting that if \(\sgn(p)=1\), we have \(\forall q \in U_0 \cap V_0,\sgn(q)=1\), and so is \(\sgn(p)=-1\).
122+
So \(\sgn\) is continuous. So \(\sgn\) is constant because \(M\) is connected.
123+
Easy to check that \(\sgn(p)=1 \iff \mathcal{B}=\mathcal{D}\), and \(\sgn(p)=-1 \iff \mathcal{C}=\mathcal{D}\).
124+
So there is only two oriention of \(M\).
125+
\end{solution}
126+
127+
\begin{problem}\label{pro:1.4}
128+
Let \(S^n(a):=\{(x^{(1)},\cdots,x^{(n + 1)} )\in \mathbb{R}_1^{n+1} :\sum_{k=1}^{n+1} (x^{(k)})^2=a^2\}\) be the ball in \(\mathbb{R}^{n+1}\) with radius \(a>0\).
129+
Let \(S:=(0,\cdots,-a),N:=(0,\cdots,a)\) be the South Pole and North Pole respectively.
130+
Let \(U_+=S^n(a)\setminus\{S\},U_-=S^n(a)\setminus\{N\}\).
131+
Let \(\phi_+:U_+ \to \mathbb{R}^n,\phi_-:U_- \to \mathbb{R}^n\).
132+
\((\xi^{(1)},\cdots,\xi^{(n)})=\phi_+(x^{(1)},\cdots,x^{(n+1)}):=\left(\frac{ax^{(1)}}{a+x^{(n+1)}},\cdots,\frac{ax^{(n)}}{a+x^{(n+1)}}\right)\).
133+
\((\eta^{(1)},\cdots,\eta^{(n)})=\phi_+(x^{(1)},\cdots,x^{(n+1)}):=\left(\frac{ax^{(1)}}{a-x^{(n+1)}},\cdots,\frac{ax^{(n)}}{a-x^{(n+1)}}\right)\).
134+
Calculate the inverses of \(\phi_+\) and \(\phi_-\), thus prove \(\{(U_+,\phi_+),(U_-,\phi_-)\}\) gives a smooth structuction of \(S^n(a)\).
135+
\end{problem}
136+
%TODO:Calculate this out.
137+
\begin{solution}
138+
Noting \(\xi^{(i)}=\frac{a x^{(i)}}{a+x^{(n+1)}}\), so \(x^{(i)}=\frac{\xi^{(i)}(a+x^{(n+1)})}{a}\).
139+
And \(\sum_{k=1}^{n+1} (x^{(k)})^2=a^2\), so \(\sum_{k=1}^{n} \frac{(\xi^{(i)})^2 (a+x^{(n+1)})^2}{a^2} + (x^{(n+1)})^2 = a^2\).
140+
\end{solution}
141+
\begin{problem}\label{pro:1.5}
142+
\end{problem}
143+
\end{document}
144+

RiemannGeometry/localcommands

Whitespace-only changes.

RiemannGeometry/template

+18
Original file line numberDiff line numberDiff line change
@@ -0,0 +1,18 @@
1+
%!Mode:: "TeX:UTF-8"
2+
%!TEX TS-program = xelatex
3+
\documentclass{ctexart}
4+
\newif\ifpreface
5+
\prefacetrue
6+
\input{../../../global/all}
7+
\begin{document}
8+
\large
9+
\setlength{\baselineskip}{1.2em}
10+
\ifpreface
11+
\input{../../../global/preface}
12+
\else
13+
\maketitle
14+
\fi
15+
\newgeometry{left=2cm,right=2cm,top=2cm,bottom=2cm}
16+
%from_here_to_type
17+
18+
\end{document}

0 commit comments

Comments
 (0)